Download as pdf or txt
Download as pdf or txt
You are on page 1of 2

Due December 5, 2017

Quantum Mechanics, exercise 03


1 Probability current
The definition for probability current is: J~ = − 2m
ih̄
(Ψ∗ ∇Ψ − Ψ∇Ψ∗ )

1. Show that J~ can be written as J~ = h̄ ∗


m Im (Ψ ∇Ψ).

2. The wavefunction can always be represented as Ψ(~r) = R(~r)eiφ(~r) , where R and φ are real
functions. Get an expression for J~ in terms of R and φ.
~
3. Show that addition of a constant phase to the wavefunction does not affect J.

4. Show that if the phase φ(~r) is constant in space, J~ is zero.

5. Show that J~ of a purely real wavefunction is zero.


6. For the potential step shown in class, in the case E > V0 : calculate the probability current for
x < 0 and x > 0. Show that R+T=1.

2 Delta-well as a limit of a finite well


A particle of mass m is bound to the potential V (x) = −W0 δ(x) with W0 > 0. In this question we
address the delta-potential as a limiting case of a finite well problem. In the following, you can rely
on the solution for a finite well of depth V0 and width L solved in class.

1. For which limiting values of V0 and L the finite well potential reaches the delta-potential? Express
W0 in terms of V0 and L.
2. In the limit found in (1), find and write explicitly the symmetrical wavefunctions of the system.
3. Bonus: In the limit found in (1), show there is only one symmetrical solution, with the eigenvalue
2
E = − m(V 0 L)
2h̄2
, and no anti-symmetrical solutions.

3 Step-Delta well
Consider a one-dimensional potential with a step function component and an attractive delta function
component just at the edge, namely:

h̄2 g 2
V (x) = V Θ(x) − δ(x),
2m
where 
1 x≥0
Θ(x) =
0 x<0

1. Compute the reflection coefficient for particles incident from the left with energy E > V .
2. Consider the case E < 0 and determine the energy eigenvalues and eigenfunctions of any existing
bound-state solutions.

1
4 Infinite potential well - properties (1)
A particle occupies the ground state of an infinite quantum well, with boundaries at x = ±a/2.
Suddenly, the boundaries are moved to be at x = ±b/2, with b > a.
1. What is the probability that the particle occupies the ground state of the new well?
2. What is the probability for it to be in the 1st excited state?

3. Find the momentum distribution function a(p). Draw |a(p)|2 and explain your result. In partic-
ular, explain why it does not diverge.

5 Infinite potential well - properties (2)


For the eigenstates of an infinite quantum well with boundaries at x = ±L/2, calculate:
p
1. The expectation value hxi and standard deviation ∆x = hx2 i − hxi2 .
p
2. The expectation value hpi and standard deviation ∆p = hp2 i − hpi2 .
3. The measure of uncertainty ∆x∆p.
4. Relying on (3), what happens at very high eigenstates?

6 Wavefunction properties
2
/σ 2 ip0 x/h̄
Consider the following wavefunction: Ψ(x) = CN e−x e .
1. Assuming Ψ(x) is normalized, calculate CN .

2. Calculate the momentum distribution a(p) of this wavefunction.


p
3. Calculate hxi, hx2 i, ∆x = hx2 i − hxi2 .
p
4. Calculate hpi, hp2 i, ∆p = hp2 i − hpi2 .
5. Calculate the uncertainty in this case, namely ∆x∆p.

You might also like